Today’s bonus weekend question will test your ability to recognize the diagnostics of a commonly tested USMLE topic.
I’m Paul Ciurysek, MD, founder of The USMLE Guys, and this is The USMLE Times. This daily newsletter aims to provide super high-yield USMLE concepts commonly tested on exam day. All content is FREE! If you’d like to work with me and my team, please see the options at the bottom of today’s newsletter. Please share the newsletter with a friend if you’d like to support our efforts!
A 32-year-old woman presents to her primary care physician with complaints of progressive muscle weakness and fatigue. She reports that her symptoms are worse at the end of the day and improve with rest. She first noticed difficulty keeping her eyes open while reading in the evenings, and recently she has experienced trouble chewing and swallowing, particularly during dinner. She denies any pain, fever, or recent illnesses. Physical examination reveals bilateral ptosis that worsens with sustained upward gaze and mild weakness in her proximal limb muscles. A laboratory test used to demonstrate the presence of antibodies directed against a specific component of the neuromuscular junction is negative. Which of the following additional laboratory findings, if present, would most strongly support the suspected diagnosis?
A) Presence of antinuclear antibodies with a speckled pattern
B) Antibodies against voltage-gated calcium channels
C) Antibodies against titin and ryanodine receptors
D) Elevated serum levels of thyroid-stimulating hormone
E) Antibodies against muscle-specific kinase
Detailed Breakdown of Answers + Correct Answer Below ⏬
ANSWER + QUESTION BREAKDOWN
It’s important to adopt the correct MENTAL MODEL when answering USMLE questions; it saves time and increases accuracy. The mental model outlined below is a foundational component of our test-taking skills masterclass (check it out if you want to elevate your skills). Here’s how to think through this question:
Step 1. Read the last line to get to the heart of the question: “Which of the following additional laboratory findings, if present, would most strongly support the suspected diagnosis?”
Step 2: Is this a first-, second-, or third-order question?
Answer: 3rd order. 1st: Identify the condition in question; 2nd: Identify the associated laboratory finding.
Step 3: Read the vignette carefully and ask yourself: “Based on my diagnosis, the lab finding most likely to support it is ____________________”
Step 4. Look at the answer choices and select the option most closely resembling your final thought from “Step 3” above.
GENERAL ANALYSIS
The 32-year-old woman presents with progressive muscle weakness and fatigue that worsens throughout the day, along with bilateral ptosis, difficulty chewing and swallowing, and proximal limb muscle weakness. These findings are characteristic of myasthenia gravis, an autoimmune disorder affecting the neuromuscular junction.
ANSWER CHOICES:
CHOICE A: Presence of antinuclear antibodies with a speckled pattern
Explanation: ANA positivity is associated with autoimmune diseases such as systemic lupus erythematosus (SLE). SLE does not typically cause progressive muscle weakness that worsens with activity or ptosis. ANA testing is not relevant to diagnosing MG.
CHOICE B: Antibodies against voltage-gated calcium channels
Explanation: Antibodies against voltage-gated calcium channels are seen in Lambert-Eaton myasthenic syndrome (LEMS), a paraneoplastic syndrome often associated with small cell lung cancer.
CHOICE C: Antibodies against titin and ryanodine receptors
Explanation: Antibodies against titin and ryanodine receptors are associated with late-onset MG or thymoma-associated MG. While these antibodies can occur in some forms of MG, they are not diagnostic for seronegative MG.
CHOICE D: Elevated serum levels of thyroid-stimulating hormone
Explanation: Elevated TSH indicates hypothyroidism, which can cause fatigue and muscle weakness. Hypothyroidism does not explain the hallmark features of MG, such as ptosis, worsening weakness with activity, and improvement with rest.
CHOICE E: Antibodies against muscle-specific kinase
Explanation: MuSK is a protein critical for clustering acetylcholine receptors at the neuromuscular junction. Antibodies against MuSK disrupt this process, leading to impaired neuromuscular transmission and symptoms of MG.
ANSWER & KEY CONCEPTS…
E) Antibodies against muscle-specific kinase (MuSK)
Myasthenia gravis (MG) is caused by autoantibodies that interfere with neuromuscular transmission by targeting components of the neuromuscular junction.
In seropositive MG, antibodies target the acetylcholine receptor (AChR). However, in about 10-15% of cases, patients are seronegative for AChR antibodies but may have antibodies against muscle-specific kinase (MuSK).
MuSK is a protein critical for clustering acetylcholine receptors at the neuromuscular junction. Antibodies against MuSK disrupt this process, leading to impaired neuromuscular transmission and symptoms of MG.
The presence of MuSK antibodies strongly supports the diagnosis of seronegative MG.